home

Exercise 1.2.1.

Solution (a):

Assume, for contradiction, that there exist integers \(p\) and \(q\) such that \(\gcd(p,q) =1 \) and \[ \Bigg( \, \frac{p}{q} \, \Bigg)^2 = 3 \tag{1}\] Next, equation (1) implies \[p^2 = 3q^2 \tag{2} \] This shows that \(p^2\) is a multiple of 3. Now, the only way for a perfect square to be a multiple of 3 is if its factors also include 3. Therefore, \(p\) must also be a multiple of 3. Let \(p=3k\), where \(k\) is an integer. Substituting this into (2) gives: \[ (3k)^2 = 3q^2 \] \[ 9k^2 = 3q^2 \] \[ 3k^2 = q^2 \tag{3} \] This shows that \(q^2\) is a multiple of 3, and hence, \(q\) is a multiple of 3. This contradicts our hypothesis that \(\gcd(p,q) = 1\). From this reasoning, we can only assume that equation (1) cannot hold for any integers \(p\) and \(q\). Thus, \(\sqrt{3}\) is irrational.

Q.E.D.

Yes, a similar argument works to show that \(\sqrt(6)\) is irrational. We can replace equation (2) with \[ p^2 = 6q^2 \tag{4} \] which shows that \(p\) is a multiple of 6. Let \(p = 6k \) and substitute into (4). \[ (6k)^2 = 6q^2 \] \[ 36k^2 = 6q^2 \] \[ 6k^2 = q^2 \] This tells us that \(q\) is a multiple of 6. By the same reasoning as above, \(\sqrt{6}\) is irrational.

Solution (b):

From \(p^2 = 4q^2\), we can only conclude that \(p\) is even.

arrow_forward